Unit III

¡Supera tus tareas y exámenes ahora con Quizwiz!

Doppler Blood Flow Analysis ( Umbilical velicometry)

*When: 16-18 weeks Why: studies blood flow in high risk pregnancies How: ultrasound Results: Persistent high ratios consistent with intrauterine growth restriction

Daily fetal movement counts

*When:anytime Why: monitors condition of fetus How: mother lies still and counts how many times baby moves in an hour, needs to be done at time of day when baby is most active Results: <3 movements in an hour needs further testing

Uterine Rupture is

IMMEDIATE HYSTERECTOMY

Accelerations OK

IT IS OK

_________ have to be occurring to separate placenta

Contractions

Promotion of Comfort During the 2nd Stage

Cool cloths Encourage rest Sips and chips Position of comfort

signs of third stage

Globular shaped uterus Rise in the fundus Sudden gush of blood Lengthening of the cord

DIFFERNCE IN A SPINAL AND AN EPIDURAL

SPINAL leaves nothing in the back, EPIDURAL still has a catheter in the spine area

Sc

Scapula

1) Passageway

True and false pelvis

VBAC

Vaginal birth after a cesarean section- 60-80% of women can vaginally deliver after a C-section; VBAC can be a common cause of Rupture of the Uterus

pulling on the cord.....

Very dangerous. Can invert the uterus and will need emergency hysterectomy.

Complete

WORST kind because the placenta completely detaches from the wall and there is massive bleeding

Burst of energy

all of the above brings a burst of enegry on. This is called nesting

anthropid

anthropid apes

lightening

baby has lightened off of diaphragm (occurs about 2 weeks before term

CPD- Cephalopelvic Disproportion

baby is to big and C-Section is required

Occiput

back of the head

platypelloid

can not deliver vaginally

Upright position

gravity

COMFORT MEASURES

heating pads, warm blankets, Doula applying pressure at pressure points

Biggest difference between true and false labor

progression in dilation and effacement

Variable Cord

quick response to cord compression- V shaped HR lines

Malposition

related to occipitoposterior posterior (ROP or LOP) baby is turned backwards, mother will complain of back pain. Mother changes position trying to get the fetus to flip or move. May try manual moving of the fetus if the dilation is enough

1st degree tear-

skin and structure superficial to the muscles are torn

Fetal lie: +Longitudinal or vertical

spines are parallel

ENGAGED

the first part of the body has passed through the first part of the pelvis. This occurs in the week before delivery.

Accelerations

the increase is 15 b/m or greater and lasts 15 sec. or more with a return to the baseline < 2 minutes from the beginning

+4 =

they are delivered +4 on the floor

intensity

how strong was it at it's peak?

Bony pelvis

separated by the brim into 2 parts; the false (above the brim) and true pelvis (3 planes-the inlet, the cavity, and the outlet

MEDIAN

slices up and down the vagina towards the rectum

Fetal lie: +transverse or horizontal

spines are perpendicular

Process of Labor

"lightening" breathe easier bladder pressure low backache bloody show (brown or blood tinged cervical mucous) cervical softening or ripening weight loss burst of energy Braxton Hicks contractions Rupture of Membranes

There are four clients in active labor in the labor suite. Which of the womenshould the nurse monitor carefully for the potential of uterine rupture? 1.Age 15, G3P0020, in active labor. 2.Age 22, G1P0000, eclampsia. 3.Age 25, G4P3003, last delivery by cesarean section. 4.Age 32, G2P0100, first baby died during labor

...3.Age 25, G4P3003, last delivery by cesarean section

A woman has just arrived at the labor and delivery suite. In order to report theclient's status to her primary health care practitioner, which of the following assess-ments should the nurse perform? Select all that apply. 1.Fetal heart rate. 2.Contraction pattern. 3.Contraction stress test. 4.Vital signs. 5.Biophysical profile

1.Fetal heart rate. 2.Contraction pattern. 4.Vital signs.

A woman had a baby by normal spontaneous delivery 10 minutes ago. The nursenotes that a gush of blood was just expelled from the vagina and the umbilical cordlengthened. What should the nurse conclude? 1.The woman has an internal laceration. 2.The woman is about to deliver the placenta. 3.The woman has an atonic uterus. 4.The woman is ready to expel the cord bloods

2.The woman is about to deliver the placenta.

A client had an epidural inserted 2 hours ago. It is functioning well, the client is hemodynamically stable, and the client's labor is progressing as expected. Which of the following assessments is highest priority at this time? 1.Assess blood pressure every 15 minutes. 2.Assess pulse rate every 1 hour. 3.Palpate bladder. 4.Ausculate lungs.

3.Palpate bladder....

A nurse is monitoring a client who is receiving an amnioinfusion. Which of the fol-lowing assessments is critical for the nurse to make in order to prevent a serious complication related to the procedure? 1.Color of the amniotic fluid. 2.Maternal blood pressure 3.Cervical effacement. 4.Uterine resting tone

4.Uterine resting tone...

Occult

hidden cord, not hanging out of the vagina

Powers or forces

how body acts to labor

A known drug addict is in active labor. She requests pain medication. Which of the following actions by the nurse is appropriate? 1.Encourage the woman to refrain from taking medication to protect the fetus. 2.Notify the physician of her request. 3.Advise the woman that she can receive only an epidural because of her history. 4.Assist the woman to do labor breathing

...2.Notify the physician of her request

A full-term client, contracting every 15 min x 30 sec, has had ruptured membranes for 20 hours. Which of the following nursing interventions is contraindicated at this time? 1.Intermittent fetal heart auscultation. 2.Vaginal examination. 3.Intravenous fluid administration. 4.Nipple stimulation

...2.Vaginal examination.

During a vaginal delivery, the obstetrician declares that a shoulder dystocia has oc-curred. Which of the following actions by the nurse is appropriate at this time? 1.Administer oxytocin intravenously per doctor's orders. 2.Flex the woman's thighs sharply toward her hips. 3.Apply oxygen using a tight-fitting face mask. 4.Apply downward pressure on the woman's fundus

2.Flex the woman's thighs sharply toward her hips...

A client, who is 7 cm dilated and 100% effaced, is breathing at a rate of 30 breathsper minute during contractions. Immediately after a contraction, she complains of tingling in her fingers with some lightheadedness. Which of the following actions should the nurse take at this time? 1.Assess the blood pressure. 2.Have the woman breathe into a bag. 3.Turn the woman onto her side. 4.Check the fetal heart rate

2.Have the woman breathe into a bag.

A woman is in active labor and is being monitored electronically. She has just re-ceived Stadol 2 mg IM for pain. Which of the following fetal heart responses would the nurse expect to see on the internal monitor tracing? 1.Variable decelerations. 2.Late decelerations. 3.Decreased variability. 4.Transient acceleration

3.Decreased variability.

On vaginal examination, it is noted that a woman with a well-functioning epiduralis in the second stage of labor. The station is -2 and the baseline fetal heart rate is130 with no decelerations. Which of the following nursing actions is appropriate at this time? 1.Coach the woman to hold her breath while pushing 3 to 4 times with each contraction. 2.Administer oxygen via face mask at 8 to 10 liters per minute. 3.Delay pushing until the baby descends further and the mother has a strong urge to push. 4.Place the woman on her side and assess her oxygen saturation

3.Delay pushing until the baby descends further and the mother has a strong urge to push.

A client with an internal fetal monitor catheter in place has just received IV butor-phanol (Stadol) for pain relief. Which of the following monitor tracing changes should the nurse anticipate? 1.Early decelerations. 2.Late decelerations. 3.Diminished short- and long-term variability 4.Accelerations after contractions.

3.Diminished short- and long-term variability...

A woman with severe preeclampsia, 38 weeks' gestation, is being induced with IV oxytocin (Pitocin). Which of the following would warrant the nurse to stop the infusion? 1.Blood pressure 160/110. 2.Frequency of contractions every 3 minutes. 3.Duration of contractions of 120 seconds. 4.Fetal heart rate 156 with early decelerations.

3.Duration of contractions of 120 seconds...

Which of the following responses is the primary rationale for providing general information as well as breathing and relaxation exercises in childbirth education classes? 1.Mothers who are doing breathing exercises during labor will refrain from yelling. 2.Breathing and relaxation exercises are less exhausting than crying and moaning. 3.Knowledge learned at childbirth education classes helps to break the fear-tension-pain cycle. 4.Childbirth education classes help to promote positive maternal-newborn bonding

3.Knowledge learned at childbirth education classes helps to break the fear-tension-pain cycle.

When performing Leopold's maneuvers, the nurse notes that the fetus is in the left occiput anterior position. Where should the nurse place a fetoscope best to hearthe fetal heart beat? 1.Left upper quadrant. 2.Right upper quadrant. 3.Left lower quadrant. 4.Right lower quadrant.

3.Left lower quadrant.

The nurse is caring for a nulliparous client who attended Lamaze childbirth educa-tion classes. Which of the following techniques should the nurse include in her plan of care? Select all that apply. 1.Hypnotic suggestion. 2.Rhythmic chanting. 3.Muscle relaxation. 4.Pelvic rocking. 5.Abdominal massage

3.Muscle relaxation. 4.Pelvic rocking. 5.Abdominal massage

The nurse enters a laboring client's room. The client is complaining of intense back pain with each contraction. The nurse concludes that the fetus is likely in which of the following positions? 1.Mentum anterior. 2.Sacrum posterior. 3.Occiput posterior. 4.Scapula anterior.

3.Occiput posterior

The nurse is providing acupressure to provide pain relief to a woman in labor. Where is the best location for the acupressure to be applied? 1.On the malleolus of the wrist. 2.Above the patella of the knee. 3.On the medial aspect of the lower leg. 4.Below the medial epicondyle of the elbow

3.On the medial aspect of the lower leg.

A 29-week-gravid client is admitted to the labor and delivery unit with vaginal bleeding. To differentiate between placenta previa and abruptio placentae, the nurse should assess which of the following? 1.Leopold's maneuver results. 2.Quantity of vaginal bleeding. 3.Presence of abdominal pain. 4.Maternal blood pressure

3.Presence of abdominal pain... The most common difference between placenta previa and placenta abruption is the absence or presence of abdominal pain

A client is complaining of severe back labor. Which of the following nursing inter- ventions would be most effective? 1.Assist mother with childbirth breathing. 2.Encourage mother to have an epidural. 3.Provide direct sacral pressure. 4.Use a hydrotherapy tub

3.Provide direct sacral pressure.

The childbirth educator is teaching a class of pregnant couples the breathing technique that is most appropriate during the second stage of labor. Which of the fol-lowing techniques did the nurse teach the women to do? 1.Alternately pant and blow. 2.Take rhythmic, shallow breaths. 3.Push down with an open glottis. 4.Do slow chest breathing.

3.Push down with an open glottis.

A nurse has just performed a vaginal examination on a client in labor. The nurse palpates the baby's buttocks as facing the mother's right side. Where should thenurse place the external fetal monitor electrode? 1.Left upper quadrant (LUQ). 2.Left lower quadrant (LLQ). 3.Right upper quadrant (RUQ). 4.Right lower quadrant (RLQ)

3.Right upper quadrant (RUQ).

During the delivery of a macrosomic baby, the woman develops a fourth-degreelaceration. How should the nurse document the extent of the laceration in the woman's medical record? 1.Into the musculature of the buttock. 2.Through the urinary meatus. 3.Through the rectal sphincter. 4.Into the head of the clitoris

3.Through the rectal sphincter....

A nurse is teaching childbirth education classes to a group of pregnant teens. Which of the following strategies would promote learning by the young women? 1.Avoiding the discussion of uncomfortable procedures like vaginal exams andblood tests. 2.Focusing the discussion on baby care rather than labor and delivery. 3.Utilizing visual aids like movies and posters during the classes. 4.Having the classes at a location other than high school to reduce their embarrassment

3.Utilizing visual aids like movies and posters during the classes.

A woman, who is in active labor, is told by her obstetrician, "Your baby is in the flexed attitude." When she asks the nurse what that means, what should the nurse say? 1.The baby is in the breech position. 2.The baby is in the horizontal lie. 3.The baby's presenting part is engaged. 4.The baby's chin is resting on its chest

4.The baby's chin is resting on its chest

A client, G3P2002, is immediately postexternal version. The nurse monitors this client carefully for which of the following? 1.Decreased urinary output. 2.Elevated blood pressure. 3.Severe occipital headache. 4.Variable fetal heart decelerations

4.Variable fetal heart decelerations...

An induction of a 42-week gravida with IV oxytocin (Pitocin) is begun at 0900 at arate of 0.5 milliunits per minute. The woman's primary physician orders: Increase the oxytocin drip by 0.5 milliunits per minute every 10 minutes until contractions are every 3 minutes x60 seconds. The nurse refuses to comply with the order. Which of the following is the rationale for the nurse's action? 1.Fetal distress has been noted in labors when oxytocin dosages greater than 2 milliunits per minute are administered. 2.The relatively long half-life of oxytocin can result in unsafe intravascular con-centrations of the drug. 3.It is unsafe practice to administer oxytocin intravenously to a woman who is carrying a postdates fetus. 4.A contraction duration of 60 seconds can lead to fetal compromise in a baby that is postmature

...2.The relatively long half-life of oxytocin can result in unsafe intravascular concentrations of the drug.

A delirious patient is admitted to the hospital in labor. She has had no prenatal careand vials of crack cocaine are found in her pockets. The nurse monitors this client carefully for which of the following intrapartal complications? 1.Prolonged labor. 2.Prolapsed cord. 3.Abruptio placentae. 4.Retained placenta

...3.Abruptio placentae.

A client, 39 weeks' gestation, fetal heart baseline at 144 bpm, tells the admitting labor and delivery room nurse that she has had to wear a pad for the past 4 days,"because I keep leaking urine." Which of the following is an appropriate action for the nurse to perform at this time? 1.Palpate the woman's bladder to check for urinary retention. 2.Obtain a urine culture to check for a urinary tract infection. 3.Assess the fluid with nitrazine and see if the paper turns blue. 4.Percuss the woman's uterus and monitor for ballottement

...3.Assess the fluid with nitrazine and see if the paper turns blue

A laboring woman, who has developed an apparent amniotic fluid embolism, is not breathing and has no pulse. In addition to calling for assistance, which of the follow-ing actions by the nurse, who is alone with the patient, is appropriate at this time? 1.Perform cardiac compressions and breaths in a 15 to 2 ratio 2.Provide chest compressions at a depth of 3 inches. 3.Compress the chest at the lower 1 ⁄ 2 of the sternum. 4.Provide rescue breaths over a 10-second time frame

...3.Compress the chest at the lower 1 ⁄ 2 of the sternum.

A woman who is hepatitis B-surface antigen positive is in active labor. Which action by the nurse is appropriate at this time? 1.Obtain an order from the obstetrician to prepare the client for cesarean delivery. 2.Obtain an order from the obstetrician to administer intravenous ampicillin during labor and the immediate postpartum. 3.Obtain an order from the pediatrician to administer hepatitis B immune globulin and hepatitis B vaccine to the baby after birth. 4.Obtain an order from the pediatrician to place the baby in isolation after delivery.

...3.Obtain an order from the pediatrician to administer hepatitis B immune globulin and hepatitis B vaccine to the baby after birth.

Which of the following nonpharmacological interventions recommended by nurse midwives may help a client at full term to go into labor? Select all that apply. 1.Engage in sexual intercourse. 2.Ingest evening primrose oil. 3.Perform yoga exercises. 4.Eat raw spinach. 5.Massage the breast and nipples.

1.Engage in sexual intercourse. 2.Ingest evening primrose oil 5.Massage the breast and nipples.

A client, 42 weeks' gestation, is admitted to the labor and delivery suite with a diagnosis of acute oligohydramnios. The nurse must carefully observe this client for signs of which of the following? 1.Fetal distress. 2.Dehydration. 3.Oliguria. 4.Jaundice.

1.Fetal distress....

A client is receiving terbutaline (Brethine) for preterm labor. Which of the following findings would warrant stopping the infusion? 1.Fetal heart rate 210 bpm. 2.Maternal heart rate 60 bpm. 3.Early decelerations. 4.Beat-to-beat variability.

1.Fetal heart rate 210 bpm... When the FH is 200 bpm, the terbutaline should be stopped.

Which of the following signs/symptoms would the nurse expect to see in a woman with abruptio placentae? 1.Increasing fundal height measurements. 2.Pain-free vaginal bleeding. 3.Fetal heart accelerations. 4.Hyperthermia with leukocytosis

1.Increasing fundal height measurements Fundal heights increase during pregnancy approximately 1 cm per week. When a placental abruption occurs, the height increases hour by hour

The nurse auscultates a fetal heart rate of 152 on a client in early labor. Which of the following actions by the nurse is appropriate? 1.Inform the mother that the rate is normal. 2.Reassess in 5 minutes to verify the results. 3.Immediately report the rate to the health care practitioner. 4.Place the client on her left side and apply oxygen by face mask.

1.Inform the mother that the rate is normal.

Which of the following choices includes the correct order of the cardinal moves of labor? 1.Internal rotation, extension, external rotation. 2.External rotation, descent, extension. 3.Extension, flexion, internal rotation. 4.External rotation, internal rotation, expulsion

1.Internal rotation, extension, external rotation.

A baby is entering the pelvis in the vertex presentation and in the extended attitude. The nurse determines that which of the following positions is consistent with this situation? 1.LMA (left mentum anterior). 2.LSP (left sacral posterior). 3.RScT (right scapular transverse). 4.ROP (right occiput posterior)

1.LMA (left mentum anterior)....

The labor and delivery nurse performs Leopold's maneuvers. A soft round mass is felt in the fundal region. A flat object is noted on the left and small objects are noted on the right of the uterus. A hard round mass is noted above the symphysis. Which of the following positions is consistent with these findings? 1.Left occipital anterior (LOA) 2.Left sacral posterior (LSP) 3.Right mentum anterior (RMA) 4.Right sacral posterior (RSP)

1.Left occipital anterior (LOA)

The nurse is assisting in the delivery of a baby via vacuum extraction. Which of the following nursing diagnoses for the gravida is appropriate at this time? 1.Risk for impaired skin integrity. 2.Risk for body image disturbance. 3.Risk for impaired parenting. 4.Risk for ineffective sexuality pattern

1.Risk for impaired skin integrity....

An obstetrician declares at the conclusion of the third stage of labor that a womanis diagnosed with placenta accreta. The nurse would expect to see which of the following signs/symptoms? 1.Hypertension. 2.Hemorrhage. 3.Bradycardia. 4.Hyperthermia

2.Hemorrhage....

The nurse is assessing a client who states, "I think I'm in labor." Which of the fol-lowing findings would positively confirm the client's belief? 1.She is contracting q 5 minx60 sec. 2.Her cervix has dilated from 2 to 4 cm. 3.Her membranes have ruptured. 4.The fetal head is engaged

2.Her cervix has dilated from 2 to 4 cm.

A nurse is assessing the vital signs of a client in labor at the peak of a contraction. Which of the following findings would the nurse expect to see? 1.Decreased pulse rate. 2.Hypertension. 3.Hyperthermia 4.Decreased respiratory rate.

2.Hypertension.

The nurse wishes to assess the variability of the fetal heart rate. Which of the following actions must precede this assessment? 1.Place the client in the lateral recumbent position. 2.Insert an internal fetal monitor electrode. 3.Administer oxygen to the mother via face mask. 4.Ask the mother to indicate when she feels fetal movement

2.Insert an internal fetal monitor electrode.

The nurse is assessing the fetal station during a vaginal examination. Which of the following structures should the nurse palpate? 1.Sacral promontory 2.Ischial spines. 3.Cervix. 4.Symphysis pubis

2.Ischial spines

During a vaginal examination, the nurse palpates fetal buttocks that are facing the left posterior and are 1 cm above the ischial spines. Which of the following is consistent with this assessment? 1.LOA -1 station. 2.LSP -1 station. 3.LMP+1 station. 4.LSA+1 station.

2.LSP -1 station

A physician has notified the labor and delivery suite that four clients will be admit-ted to the unit. The client with which of the following clinical findings would be a candidate for an external version? 1.+3 station. 2.Left sacral posterior position .3.Flexed attitude. 4.Rupture of membranes for 24 hours

2.Left sacral posterior position... LSP position is a breech presentation. It may be appropriate for a physician to perform an external version prior to this delivery

In which of the following clinical situations would it be appropriate for an obstetrician to order a labor nurse to perform amnioinfusion? 1.Placental abruption. 2.Meconium-stained fluid. 3.Polyhydramnios. 4.Late decelerations

2.Meconium-stained fluid...

The nurse is caring for an eclamptic client. Which of the following is an important action for the nurse to perform? 1.Check each urine for presence of ketones. 2.Pad the client's bed rails and head board. 3.Provide visual and auditory stimulation. 4.Place the bed in the high Fowler's position

2.Pad the client's bed rails and head board....

In addition to breathing with contractions, which of the following actions can help a woman in labor to work with the pain of the first stage of labor? 1.Lying in the lithotomy position. 2.Performing effleurage. 3.Practicing Kegel exercises. 4.Pushing with each contraction

2.Performing effleurage

A client, G2P1001, 5 cm dilated and 40% effaced, has just received an epidural. Which of the following actions is important for the nurse to make at this time? 1.Assess the woman's temperature. 2.Place a wedge under the woman's side. 3.Place a blanket roll under the woman's feet. 4.Assess the woman's pedal pulses

2.Place a wedge under the woman's side.

The fetal monitor tracing of a laboring woman who is 9 cm dilated shows recurring late decelerations to 100 bpm. The nurse notes a moderate amount of greenish colored amniotic fluid gush from the vagina after a practitioner performs an amniotomy. Which of the following nursing diagnoses is appropriate at this time? 1.Risk for infection related to rupture of membranes. 2.Risk for fetal injury related to possible intrauterine hypoxia. 3.Risk for impaired tissue integrity related to vaginal irritation. 4.Risk for maternal injury related to possible uterine rupture.

2.Risk for fetal injury related to possible intrauterine hypoxia....

The nurse is caring for a 30-week-gestation client whose fetal fibronectin (fFN)levels are positive. It is essential that she be taught about which of the following? 1.How to use a blood glucose monitor. 2.Signs of preterm labor. 3.Signs of preeclampsia. 4.How to do fetal kick counts

2.Signs of preterm labor.... Positive fetal fibronectin levels are seen in clients who deliver preterm

A physician writes the following order—administer ampicillin 1 Gm IV q 4 h until delivery—for a newly admitted laboring client with ruptured membranes. Theclient had positive vaginal and rectal cultures for group B streptococcal bacteria at 36 weeks' gestation. Which of the following is a rationale for this order? 1.The client is at high risk for chorioamnionitis. 2.The baby is at high risk for neonatal sepsis. 3.The bacterium is sexually transmitted. 4.The bacterium causes puerperal sepsis

2.The baby is at high risk for neonatal sepsis....

A woman, G1 P0000, 40 weeks' gestation, entered the labor suite stating that she isin labor. Upon examination it is noted that the woman is 2 cm dilated, 30% effaced,contracting every 12 min x30 sec. Fetal heart rate is in the 140s with good variability and spontaneous accelerations. What should the nurse conclude when re-porting the findings to the primary health care practitioner? 1.The woman is high risk and should be placed on tocolytics. 2.The woman is in early labor and could be sent home. 3.The woman is high risk and could be induced. 4.The woman is in active labor and should be admitted to the unit.

2.The woman is in early labor and could be sent home.

Three 30-week-gestation clients are on the labor and delivery unit in preterm labor. For which of the clients should the nurse question a doctor's order for betaagonist tocolytics? 1.A client with hypothyroidism. 2.A client with breast cancer. 3.A client with cardiac disease. 4.A client with asthma

3.A client with cardiac disease... A history of cardiac disease would place a client who is to receive a beta agonist medication at risk. The nurse should question this order

A woman being induced with oxytocin (Pitocin) is contracting every 3 min x 30 seconds. Suddenly the woman becomes dypsneic, cyanotic, and begins to cough up bloody sputum. Which of the following nursing interventions is of highest priority? 1.Check blood pressure. 2.Assess fetal heart rate. 3.Administer oxygen. 4.Stop oxytocin infusion

3.Administer oxygen.

A woman has been in the second stage of labor for 2 1 /2 hours. The fetal head is at +4 station and the fetal heart is showing mild late decelerations. The obstetrician advises the woman that the baby will be delivered with forceps. Which of the following actions should the nurse take at this time? 1.Obtain a consent for the use of forceps. 2.Encourage the woman to push between contractions. 3.Assess the fetal heart rate after each contraction. 4.Advise the woman to refuse the use of forceps

3.Assess the fetal heart rate after each contraction....

A nurse is caring for a gravid client who is G1P0000, 35 weeks' gestation. Which of the following would warrant the nurse to notify the woman's health care practi-tioner that the client is in preterm labor? 1.Contraction frequency every 15 minutes. 2.Effacement 10%. 3.Dilation 3 cm. 4.Contraction duration of 30 seconds

3.Dilation 3 cm...

The nurse is caring for two post-cesarean section clients in the postanesthesia suite.One of the clients had her surgery under spinal anesthesia, while the other client had her surgery under epidural anesthesia. Which of the following is an important difference between the two types of anesthesia that the nurse should be aware of? 1.The level of the pain relief is lower in spinals .2.Placement of the needle is higher in epidurals. 3.Epidurals do not fully sedate motor nerves. 4.Spinal clients complain of nausea and vomiting

3.Epidurals do not fully sedate motor nerves....

The labor nurse has just received shift report on four gravid patients. Which of the patients should the nurse assess first? 1.G5P2202, 32 weeks, placenta previa, today's hemoglobin 11.6 gm/dL. 2.G2P0101, 39 weeks, type 2 diabetic, blood glucose (15 minutes ago) 85 mg/dL. 3.G1P0000, 32 weeks, placental abruption, fetal heart (15 minutes ago) 120 bpm. 4.G2P1001, 39 weeks, Rh negative, today's hematocrit 31%.

3.G1P0000, 32 weeks, placental abruption, fetal heart (15 minutes ago) 120 bpm....A placental abruption is a lifethreatening situation for the fetus. It has been 15 minutes since the fetal heart was assessed. This is the nurse's priority

After a multiparous woman has been in active labor for 15 hours, an ultrasound is done. The results state that the obstetric conjugate is 10 cm and the subocipito-bregmatic diameter is 10.5 cm. Which of the following labor findings is related to these results? 1.Full dilation of the cervix. 2.Full effacement of the cervix. 3.Station of - 3. 4.Frequency every 5 minutes.

3.Station of - 3

After analyzing an internal fetal monitor tracing, the nurse concludes that there is moderate short-term variability. Which of the following interpretations should the nurse make in relation to this finding? 1.The fetus is becoming hypoxic. 2.The fetus is becoming alkalotic. 3.The fetus is in the middle of a sleep cycle. 4.The fetus has a healthy nervous system.

4.The fetus has a healthy nervous system.

A pregnant woman, G3P2002, had her two other children by cesarean section. Which of the following situations would mandate that this delivery also be by cesarean? 1.The woman refuses to have a regional anesthesia. 2.The woman is postdates with intact membranes. 3.The baby is in the occiput posterior position. 4.The previous uterine incisions were vertical.

4.The previous uterine incisions were vertical.... The presence of vertical incisions in the uterine wall is an absolute indication for a cesarean delivery.

An anesthesiologist informs the nurse that a woman for cesarean section will havethe procedure under general anesthesia with postoperative patient-controlled anal-gesia rather than under continuous epidural infusion. Which of the following would warrant this decision? 1.The woman has a history of drug addiction. 2.The woman is allergic to morphine sulfate. 3.The woman is a thirteen-year-old adolescent. 4.The woman has had surgery for scoliosis

4.The woman has had surgery for scoliosis...

A woman, G2 P0101, 5 cm dilated and 30% effaced, is doing first-level Lamazebreathing with contractions. The nurse detects that the woman's shoulder and facemuscles are beginning to tense during the contractions. Which of the following in-terventions should the nurse perform first? 1.Encourage the woman to have an epidural .2.Encourage the woman to accept intravenous analgesia 3.Assist the woman in changing position. 4.Urge the woman to perform the next level breathing

4.Urge the woman to perform the next level breathing

A woman who states that she "thinks" she is in labor enters the labor suite. Which of the following assessments will provide the nurse with the most valuable information regarding the client's labor status? 1.Leopold's maneuvers. 2.Fundal contractility. 3.Fetal heart assessment. 4.Vaginal examination

4.Vaginal examination

FETAL WELL-BEING test

hydramnios, oligohydramnios. Done at any time

3rd degree tear-

if it extends through the anal sphincter muscle

2nd degree tear-

if the tear extends through the muscle

RED BLOODY BALL IS ___________

inverted uterus

Signs of separation

rise in fundus of the uterus, a sudden gush of blood, and a lengthening of the cord

Voluntary bearing down by the woman creates ___________.

secondary forces

Doula

someone who is paid by you to help and support you through labor. Support system for a mother who doesn't have one.

LATENT PHASE

0-3 cm of dilation and takes 6-8 hrs to complete. Contractions are mild to moderate, 3-30 minutes apart, lasting 20-40 seconds. Assessment: doing assessment every 30-60 mins. Behavior: smiling and talkative

7 cardinal movements

1. Baby must get in position to begin descent 2. Decent through the pelvis (sac must be ruptured and the pelvis must be the right shape for baby to decend) 3. Chin must tuck in (FLEXION) in order to get his head through the pelvic canal 4. Next movement is internal rotation to get his shoulders through 5. Next he must Extends his head to get it out 6. Then he must rotate again 7. Then the hips can come out, face should be up at this point after hips come through

Mechanism of birth

1. Birth of head then shoulders then body and extremities 2. Suction 3. Cord Clamp 4. Dry and warm

3 types of Abruptio Placentae

1. Marginal 2. Central 3. Complete

The nurse is admitting a 38-week-gestation client in labor. The nurse is unable to find the fetal heart beat with a Doppler. Which of the following comments by the nurse would indicate that the nurse is in denial? 1."I'll keep trying until I find the heart beat." 2."I am sure it is the machine. If I change the battery, I'm sure it will work." 3."I am so sorry. I am not able to find your baby's heart beat." 4."Sometimes I really hate these machines."

1."I'll keep trying until I find the heart beat."...

A client in labor is talkative and happy. How many centimeters dilated would a maternity nurse suspect that the client is at this time? 1.2 cm. 2.4 cm. 3.8 cm. 4.10 cm.

1.2 cm

The nurse in the obstetrician's office is caring for four 25-week-gestation white prenatal clients who are carrying singleton pregnancies. With which of the following clients should the nurse carefully review the signs and symptoms of preterm labor (PTL)? 1.38-year-old registered nurse in an abusive relationship. 2.32-year-old secretary whose first child was born at 42 weeks' gestation. 3.26-year-old attorney whose baby has a two-vessel cord. 4.20-year-old college student with a history of long menstrual periods.

1.38-year-old registered nurse in an abusive relationship... This client is high risk for PTL because she is over 35 years of age and in an abusive relationship

The nurse is performing a vaginal examination on a client in labor. The client is found to be 5 cm dilated, 90% effaced, and station-2. Which of the following has the nurse palpated? 1.Thin cervix. 2.Bulging fetal membranes. 3.Head at the pelvic outlet. 4.Closed cervix.

1.Thin cervix.

A client enters the labor and delivery suite stating that she thinks she is in labor. Which of the following information about the woman should the nurse note from the woman's prenatal record before proceeding with the physical assessment? Select all that apply. 1.Weight gain. 2.Ethnicity and religion. 3.Age. 4.Type of insurance. 5.Gravidity and parity.

1.Weight gain. 2.Ethnicity and religion. 3.Age. 5.Gravidity and parity.

**Chorionic Villus Sampling (CVS)**

10-12 weeks

anterior fontanelle should close within

12-18 mos.

The nurse is monitoring a woman, G2P1001, 41 weeks' gestation, in labor. A 12 p.m.assessment revealed: cervix, 4 cm; 80% effaced; x 3 station; and FH 124 with moderate variability. A 5 p.m. assessment: cervix, 6 cm; 90% effaced; x 3 station; and FH120 with moderate variability. A 10 p.m. assessment: cervix, 8 cm; 100% effaced; x 3 station; and FH 124 with moderate variability. Based on the assessments, which of the following should the nurse conclude? 1.Labor is progressing well. 2.The woman is likely carrying a macrosomic fetus. 3.The baby is in fetal distress. 4.The woman will be in second stage in about five hours.

2.The woman is likely carrying a macrosomic fetus....

molding should go away within

3 days

Want the baby to be engaged at

35-36 weeks

Stages of Labor

4 stages of labor 1st-onset of regular contractions to full dilation of cervix>> 3 phases 2nd-full dilation to birth...cervix can no longer be felt 3rd-birth to expulsion of the placenta 4th-up to 2 hours after the delivery of the placenta...hormones changing

The nurse sees the fetal head through the vaginal introitus when a woman pushes. The nurse, interpreting this finding, tells the client, "You are pushing very well." In addition, the nurse could also state which of the following? 1."The baby's head is engaged." 2."The baby is floating." 3."The baby is at the ischial spines." 4."The baby is almost crowning."

4."The baby is almost crowning."

Upon examination, a nurse notes that a woman is 10 cm dilated, 100% effaced, and-3 station. Which of the following actions should the nurse perform during the next contraction? 1.Encourage the woman to push. 2.Provide firm fundal pressure. 3.Move the client into a squat. 4.Assess for signs of rectal pressure

4.Assess for signs of rectal pressure

Factors Affecting Labor

5 P's: Passenger, Passageway, Powers (Forces), Position of the Laboring Woman, Psyche

An insulin-dependent diabetic is in active labor. The physician has written the fol-lowing order: Administer regular insulin 5 units per hour via IV pump. The insulinhas been diluted as follows: 50 units/500 mL normal saline. At what rate should thenurse set the pump? __________ mL/hr

50 mL/hr... 50 units/500 mL 5 units/x mL 50 x 5 500 50 x 2500 x 50 mL/hr

moderate variability

6-25 bpm

posterior fontanelle should close

6-8 weeks

Medications should be administered during _______ phase of _______ stage.

Active phase of first stage

Episiotomy

An incision that is made to enlarge the vaginal outlet to prevent lacerations and tears.

Late Placenta

BAD NEWS BEARS, this can show IMPENDING DEATH due to a perfusion problem. Placental abruption, knot in the cord INTERVENTION: turn them on their LEFT SIDE (late left) put them on oxygen, make sure they have an IV for fluids

What is the baseline FHR?

Beats per minute (average 110-160)

NON REACTIVE NON STRESS TEST IS NOT GOOD and would be followed with

Biophysical profile or an ultrasound

Promotion of Comfort During the 1st Stage

Breathing techniques Effleurage and counterpressure Water therapy Acupressure Application of heat and cold Therapeutic touch Hypnosis Biofeedback Aromatherapy

Sacrum

Butt

Promotion of Comfort During the 3rd Stage of Labor

Deliver any meds. Monitor vitals Enhance attachment Fundal checks

Amniotic Fluid Embolism

EMERGENCY!!! Occurs when there are particles of debris (vernix, hair, skin cells, or meconium) that enter the maternal circulation and obstructs the pulmonary vessels. Mother and baby usually do not survive Mother demise 86% Fetus demise 50% Tear in the amniotic sac then dumps all of the shit and skin and such into the mothers vessels and kills her

Internal monitoring

Membranes have to had ruptured and the cervix has to be at least 2 cm dilated and the presenting part must be low enough in the pelvis for Internal monitoring to be possible 2 probes- 1 is a pin that goes under the fetal skin on the head and the 2nd is the contraction monitor and it just lays in the fetal cavity

Rupture of the Uterus

Most common cause is separation of the scar from a previous section, uterine trauma, and congenital uterine anomaly

Bloody show

Mucous plug drops out, brownish red blood tinge to fluid, more at risk for infection

Management of Amnitotic Fluid Embolism

O2 intubation and ventilation CPR position on side IV fluids/blood transfusion foley emergency birth

Most common and best position

Occiput Anterior

Placenta Accreta

Placenta implants itself into the myometrium (muscle) instead of the uterus and you will have a hysterectomy no matter what

Third stage

Placental separation

COOMBS TEST

RH factor compatability, and can be drawn at anytime but USUALLY first visit

Placenta Previa

Toyota Previa vans are always in the way Biggest difference between the ABRUPTION and PREVIA is Previa has no pain just uterine bleeding

Involuntary

bodies doing it, mother cannot control

Occiput

bottom back of head

__________ is a late sign of Hypoxia

bradycardia...<110 b/m

WHILE PUSHING YOU MUST _________

breathe

Lamaze

focus on chest breathing; have partner

SINGLE footling breach

foot dangling down

4th degree tear-

goes through the rectal wall

Pelvic Dystocia

misshaped pelvis that will not let the body vaginally deliver. Also car wreck or any other damage as to where the pelvis could have broken could cause problems with vaginal birth

Semirecumbant position

needs adequate body support

gynecoid

normal female pelvis

3rd stage of labor-

first few minutes to an hour after birth, this is the expulsion of the placenta, and mother feels relieved

A nurse notes a sinusoidal fetal heart pattern while analyzing a fetal heart tracing of a newly admitted client. Which of the following actions should the nurse take at this time? 1.Encourage the client to breathe with contractions .2.Notify the practitioner. 3.Increase the intravenous infusion. 4.Encourage the client to push with contractions.

.2.Notify the practitioner.:

When monitoring a fetal heart rate with moderate variability, the nurse notes V-shaped decelerations to 80 from a baseline of 120. One occurred during a con-traction while another occurred 10 seconds after the contraction and a third occurred 40 seconds after yet another contraction. The nurse interprets these findings as resulting from which of the following? 1.Metabolic acidosis. 2.Head compression. 3.Cord compression. 4.Insufficient uteroplacental blood flow

.3. .Cord compression.: The contractions described in the scenario result from cord compression (variable decelerations)

The health care practitioner orders the following medication for a laboring client: Stadol 0.5 mg IV stat for pain. The drug is on hand in the following concentration:Stadol 2 mg/mL. How many mL of medication will the nurse administer? _____ mL

0.25

A client is in labor and delivery with a diagnosis of HELLP syndrome. The nursenotes the following blood values: PT (prothrombin time) 99 sec (normal 60 to 85 sec).PTT (partial thromboplastin time)30 sec (normal 11 to 15 sec).For which of the following signs/symptoms would the nurse monitor the client? 1.Pink-tinged urine. 2.Early decelerations. 3.Patellar reflexes+1 4.Blood pressure 140/90.

1.Pink-tinged urine....

A midwife advises a mother that her obstetric conjugate is of average size. How should the nurse interpret that information for the mother? 1.The anterior to posterior diameter of the pelvis will accommodate a fetus with an average-sized head. 2.The fetal head is flexed so that it is of average diameter. 3.The mother's cervix is of average dilation for the start of labor. 4.The distance between the mother's physiological retraction ring and the fetalhead is of average dimensions

1.The anterior to posterior diameter of the pelvis will accommodate a fetus with an average-sized head.

While evaluating the fetal heart monitor tracing on a client in labor, the nursenotes that there are fetal heart decelerations present. Which of the following assessments must the nurse make at this time? 1.The relationship between the decelerations and the labor contractions. 2.The maternal blood pressure. 3.The gestational age of the fetus. 4.The placement of the fetal heart electrode in relation to the fetal position

1.The relationship between the decelerations and the labor contractions.

A nurse concludes that a woman is in the latent phase of labor. Which of the fol-lowing signs/symptoms would lead a nurse to that conclusion? 1.The woman talks and laughs during contractions. 2.The woman complains about severe back labor. 3.The woman performs effleurage during a contraction. 4.The woman asks to go to the bathroom to defecate

1.The woman talks and laughs during contractions.

The nurse is admitting four full term primigravid clients to the labor and delivery unit. The nurse requests pre-cesarean section orders from the health care practitioner for which of the clients? The client who has: 1.cervical cerclage. 2.FH 156 with beat-to-beat variability. 3.Maternal blood pressure of 90/60. 4.Full effacement

1.cervical cerclage....

1st stage of labor

1st phase is latent phase: during this phase there is more progress in effacement than decent, it usually lasts 6-8 hours, dilation of 0-3 cm, frequency is 3-30 minutes and duration is 20-40 seconds, mild to moderate. 2nd phase is active phase: this is the stage where pain medicine can be introduced. Duration is 3-6 hours, dilation 4-8 cm, frequency is 2-3 minutes and duration is 40-60 seconds, and are moderate to strong 3rd phase is the transition phase: this only lasts 20-40 minutes, they finish dilation 8-10 cm, frequency is 1-2 minutes apart, duration is 60-90 seconds, and are strong (COMPLAIN OF RECTAL PRESSURE)

Which of the following situations is considered a vaginal delivery emergency? 1.Third stage of labor lasting 20 minutes. 2.Fetal heart dropping during contractions. 3.Three-vessel cord. 4.Shoulder dystocia.

4.Shoulder dystocia.... Shoulder dystocia is an obstetric emergency

A G1P0, 8 cm dilated, is to receive pain medication. The health care practitioner has decided to order an opiate analgesic with an analgesic-potentiating medication. Which of the following medications would the nurse expect to be ordered as the analgesic-potentiating medication? 1.Seconal (secobarbital). 2.Phenergan (promethazine). 3.Benadryl (diphenhydramine). 4.Tylenol (acetaminophen)

2. Phenergan (promethazine).

The physician has ordered Prepidil (dinoprostone) for 4 gravidas at term. The nurse should question the order for which of the women? 1.Primigravida with Bishop score of 4. 2.Multigravida with late decelerations. 3.G1P0000 contracting every 20 minutes x 30 seconds. 4.G6P3202 with blood pressure 140/90 and pulse 92.

2. This client's fetus is already showing signs of fetal distress. Induction in-creases the risk of fetal injury

A client in labor, G2 P1001, was admitted 1 hour ago at 2 cm dilated and 50% effaced. She was talkative and excited at that time. During the past 10 minutes shehas become serious, closing her eyes and breathing rapidly with each contraction. Which of the following is an accurate nursing assessment of the situation? 1.The client had poor childbirth education prior to labor. 2.The client is exhibiting an expected behavior for labor. 3.The client is becoming hypoxic and hypercapnic. 4.The client needs her alpha-fetoprotein levels checked.

2.The client is exhibiting an expected behavior for labor.

The nurse is caring for an Orthodox Jewish woman in labor. It would be appropriate for the nurse to include which of the following in the plan of care? 1.Encourage the father to hold his partner's hand during labor. 2.Ask the woman if she would like to speak with her priest. 3.Provide the woman with a long-sleeved hospital gown. 4.Place an order for the woman's postpartum vegetarian diet.

3.Provide the woman with a long-sleeved hospital gown.

Engagement occurs:

36-37 weeks. If need to try and flip baby it has to be done before this point because baby will settle down into pelvis and once engaged it is locked in. +3 is crowning

A primigravid client received Prepidil (dinoprostone) for induction 8 hours ago. The Bishop score is now 10. Which of the following actions by the nurse is appropriate? 1.Perform nitrazine analysis of amniotic fluid. 2.Report abnormal findings to the obstetrician. 3.Place woman on her side. 4.Monitor for onset of labor

4, The nurse should monitor this client for the onset of labor

A nurse is coaching a woman who is in the second stage of labor. Which of the following should the nurse encourage the woman to do? 1.Hold her breath for twenty seconds during every contraction. 2.Blow out forcefully during every contraction. 3.Push between contractions until the fetal head is visible. 4.Take a slow cleansing breath before bearing down

4.Take a slow cleansing breath before bearing down

Cesarean Section

CPD or malpresentation Fetal distress Cord prolapse Dysfunctional labor Multiple gestation Birth canal obstruction Previous section

Marginal Previa

Edge of the placenta extends 2-3 cm over the OS, but does not cover it completely.

Bradley Method

Emphasize the room be completely dark, both partners get naked, women don't use meds, silence (silence, darkness, and nakedness... lol)

Dystocia

Long, difficult, or abnormal labor alteration in the characteristics of uterine contractions lack of progress in dilation lack of progress in fetal descent and expulsion.

Abruptio Placentae

Premature separation of the placenta from its implantation site Intense pain and sudden Most common risk facture- Hypertension, cocaine abuse, blunt external trauma (car wreck, abuse), smoking

Central

Separation at the middle, but the ends are connected. Blood is trapped between the placenta and the uterine wall

Narcotics

Stadol, Neubain (remember respiratory depression)

AMNIOCENTESIS

can only be done after week 16 (because there is not enough fluid). for woman over age of 35, or suspect abnormality. If they know mothers at high risk for preterm labor to make sure lungs are mature. Take huge needle and insert into numbed abdomen and withdraw fetal cells. Should self seal. Ultra sound guidance needed. LS ratio: lets you know if the fetus lungs are mature

**CHROIONIC VILLI SAMPLING**

can only be done between **10-12 weeks** . Advanced maternal age, genetic testing, infection, or metabolic disorders. Results depend on what is being looked for. Remove part of placenta. DONE ON women over the age of 35 or have a risk factor such as genetics, metabolic disorders, etc.... CAN BE DONE, transcervically and transabdominally depending on the location of the fetus

Minciput

chin

Blood loss: ________ for normal vaginal labor

200-250ml

Counterpressure

tennis ball in back

Vertex

top and back of head

Tears can be ....

anywhere, vaginal, cervical, urethra

*TRIPLE-MARKER TEST

done as early as 10 weeks. Levels being looked at Estriol, AFP, HCG. More definitive answer and can be done earlier.

if placenta has not separated after an hour...........

it is considered retained and have to go to surgery

Mentum

: Chin

Amniocentesis

*When: only after week 16 Why: >35 year old, suspected abnormality (genetic problems, fetal maturity, fetal hemolytic disease) How: ultrasound guided large needle in numb abdomen. Withdraw fetal cells and amniotic fluid Results: LS ratio- lets you know if fetal lungs are mature

The nurse turns off the oxytocin (Pitocin) infusion after a period of hyperstimulation. Which of the following outcomes indicates that the nurse's action was effective? 1.Intensity moderate. 2.Frequency every 3 minutes. 3.Duration 120 seconds. 4.Attitude flexed.

...2.Frequency every 3 minutes

A client telephones the labor and delivery suite and states, "My bag of waters just broke and it smells funny." Which of the following responses should the nurse make at this time? 1."Have you notified your doctor of the smell?" 2."The bag of waters always has an unusual smell." 3."Your labor should start pretty soon." 4."Have you felt the baby move since the membranes broke?"

...4."Have you felt the baby move since the membranes broke?" 22, p316

A nurse is caring for four clients on the labor and delivery unit. Which of the fol-lowing actions should the nurse take first? 1.Check the blood sugar of a gestational diabetic .2.Assess the vaginal blood loss of a client who is post-spontaneous abortion. 3.Assess the patellar reflexes of a client with mild preeclampsia. 4.Check the fetal heart rate of a client who just ruptured membranes

...4.Check the fetal heart rate of a client who just ruptured membranes

A client, G4P1021, has been admitted to the labor and delivery suite for inductionof labor. The following assessments have been made: Bishop score of 3, fetal heart rate of 156 with good variability and no decelerations, TPR 98.6ºF, P 88, R 20, BP 120/80, negative obstetric history. Cervidil (dinoprostone) has been inserted. Which of the following findings would warrant the removal of the prostaglandin suppository? 1.Bishop score of 10. 2.Fetal heart of 152. 3.Respiratory rate of 24. 4.Contraction frequency of 1 minute

...4.Contraction frequency of 1 minute

A client is on terbutaline (Brethine) via subcutaneous pump for preterm labor. The nurse auscultates the fetal heart rate at 100 beats per minute via Doppler. Which of the following actions should the nurse perform next? 1.Assess the maternal pulse while listening to the fetal heart rate. 2.Notify the health care provider. 3.Stop the terbutaline infusion. 4.Administer oxygen to the mother via face mask.

1.Assess the maternal pulse while listening to the fetal heart rate....

A nurse has been assigned to circulate during the cesarean section of triplets. Whichof the following actions should the nurse take before the birth of the babies? Select all that apply. 1.Count the number of sterile sponges. 2.Document the time of the first incision. 3.Notify the pediatric staff. 4.Perform a sterile scrub. 5.Assemble the sterile instruments

1, 2, and 3 are correct. 1. The circulating nurse should count the sterile sponges. This is done together with the scrub nurse. 2. The circulating nurse must document in the medical record all key events that occur during the surgery, including the time of the first incision. 3. The circulating nurse should notify the pediatric staff. There should be one resuscitation team assembled in the delivery room for each baby that will be delivered....

A client enters the labor and delivery suite. It is essential that the nurse note the woman's status in relation to which of the following infectious diseases? Select allthat apply. 1.Hepatitis B. 2.Rubeola. 3.Varicella. 4.Group B streptococcus. 5.HIV/AIDS

1, 4, and 5 are correct. 1. The client's hepatitis B status should be assessed 4. The client's group B streptococcus status should be assessed. 5. The client's HIV/AIDS status should be assessed

A labor nurse is caring for a client, 38 weeks' gestation, who has been diagnosed with symptomatic placenta previa. Which of the following physician orders should the nurse question? 1.Begin oxytocin drip rate at 0.5 millunits/min. 2.Assess fetal heart rate every 10 minutes. 3.Weigh all vaginal pads. 4.Assess hematocrit and hemoglobin

1.Begin oxytocin drip rate at 0.5 millunits/min...

Baseline Fetal Heart Rate

increase in sympathetic response causes an acceleration and the parasympathetic response controls slowing Baseline FHR is average rate during a 10 min. segment normal is 110-160 b/m

When during the latent phase of labor should the nurse assess the fetal heart pattern of a low-risk woman, G1 P0000? Select all that apply. 1.After vaginal exams. 2.Before administration of analgesics. 3.Periodically at the end of a contraction. 4.Every ten minutes. 5.Before ambulating

1.After vaginal exams. 2.Before administration of analgesics. 3.Periodically at the end of a contraction. 5.Before ambulating

To reduce possible side effects from a cesarean section under general anesthesia,clients are routinely given which of the following medications? 1.Antacids .2.Tranquilizers. 3.Antihypertensives. 4.Anticonvulsants.

1.Antacids...

A woman in active labor received Nubain (nalbuphine hydrochloride) 14 mg IV for pain relief. One-half hour later her respirations are 8 rpm. The nurse reports the respiratory rate to the physician. Which of the following medications would be appropriate for the physician to order at this time? 1.Narcan (naloxone). 2.Reglan (declopramide). 3.Benadryl (diphenhydramine). 4.Vistaril (hydroxyzine).

1.Narcan (naloxone)....

While caring for a client in the transition phase of labor, the nurse notes that the fetal monitor tracing shows average short-term and long-term variability with a baseline of 142 beats per minute (bpm). What should the nurse do? 1.Provide caring labor support. 2.Administer oxygen via face mask. 3.Change the client's position. 4.Speed up the client's intravenous

1.Provide caring labor support.

Immediately following administration of an epidural anesthesia, the nurse must monitor the mother for which of the following? 1.Paresthesias in her feet and legs. 2.Drop in blood pressure. 3.Increase in central venous pressure. 4.Fetal heart accelerations

2.Drop in blood pressure

Which of the following actions is appropriate for the nurse to perform when caring for a Chinese-speaking woman in active labor? 1.Apply heat to the woman's back. 2.Inquire regarding the woman's pain level. 3.Make sure that the woman's head is covered. 4.Accept the woman's loud verbalizations

2.Inquire regarding the woman's pain level.

The nurse is assessing an internal fetal heart monitor tracing of an unmedicated,full-term gravida who is in transition. Which of the following heart rate patterns would the nurse interpret as normal? 1.Variable baseline of 140 with V-shaped decelerations to 120 unrelated to contractions. 2.Variable baseline of 140 with decelerations to 100 that mirror each of the contractions. 3.Flat baseline of 140 with decelerations to 120 that return to baseline after the end of the contractions. 4.Flat baseline of 140 with no obvious decelerations or accelerations

2.Variable baseline of 140 with decelerations to 100 that mirror each of the contractions

A pregnant woman is discussing positioning and the use of leg stirrups for delivery with a labor nurse. In which of the following instances should the nurse provide further information to the client? 1.When the client states, "I am glad that deliveries can take place in a variety of places, including a Jacuzzi bathtub." 2.When the client says, "I heard that for doctors to deliver babies safely, it is essential to have the mother's legs up in stirrups." 3.When the client says, "During difficult deliveries it is sometimes necessary to put a woman's legs up in stirrups." 4.When the client states, "I heard that midwives often deliver their patients either in the side-lying or squatting position."

2.When the client says, "I heard that for doctors to deliver babies safely, it is essential to have the mother's legs up in stirrups."

The doctor writes the following order for a 31-week-gravid client with sympto-matic placenta previa: weigh all vaginal pads and estimate blood loss. The nurse weighs one of the client's saturated pads at 24 grams and a dry pad at 4 grams. How many milliliters (mL) of blood can the nurse estimate the client has bled? __________ mL

20 mL of blood... The nurse must remember that 1 mL of fluid weighs approximately 1 gram. The nurse can estimate, therefore, that the blood loss is: 24 4 20 mL of blood

The physician writes the following order for a newly admitted client in labor: Begin a 1000 cc IV of D5 1/2 NS at 150 cc/hr. The IV tubing states that the dropfactor is 10 gtt/cc. Calculate the drip rate._______ gtt/min

25

yellow fluid

indicates hypoxia, hemalytic disease, infection

A woman, 40 2 ⁄ 7 weeks' gestation, has had ruptured membranes for 15 hours with nolabor contractions. Her obstetrician has ordered 10 units oxytocin (Pitocin) to be diluted in 1000 mL D5 1 ⁄ 2 NS. The order reads: administer oxytocin IV at 0.5 milliunitsper min. Calculate the drip rate for the infusion pump to be programmed. __________ mL/hr

3 mL The nurse must do a number of calculationsto determine the pump drip rate in thisclient. First, the nurse must determine how many milliunits are in 1000 mL of fluid: 10 units in 1000 mL -10,000 milliunits in 1000 mL Next, the nurse must determine how many milliunits are to be infused per hour (sincepumps are always calibrated mL/hour): 0.5 millliunits per minute -30 milliunits per 60 minutes Finally, the nurse must do a ratio and pro-portion to determine the mL per hour: 10,000 milliunits/1000 mL -30 milliunits/ x mL x-3 mL/hr

During the third stage, the following physiological changes occur. Please place the changes in chronological order. 1.Hematoma forms behind the placenta. 2.Membranes separate from the uterine wall. 3.The uterus contracts firmly. 4.The uterine surface area dramatically decreases

3, 4, 1, 2

The nurse is assessing the Bishop score on a postdates client. Which of the following measurements will the nurse assess? Select all that apply. 1.Gestational age. 2.Rupture of membranes. 3.Cervical dilatation. 4.Fetal station .5.Cervical position.

3, 4, and 5 are correct... 3. Cervical dilatation is part of the Bishop score. 4. Fetal station is part of the Bishop score. 5. Cervical position is part of the Bishop score.

Previa Plan of Care

Transabdominal US - The placenta previa can move Pelvic rest- no sex, no tampons, no douching Term, in labor --> C-section < 36 weeks, not in labor --> rest IVF's and PRBC's Monitor FHT's

An obstetrician is performing an amniotomy on a gravid woman in transition. Which of the following assessments must the nurse make immediately following the procedure? 1.Maternal blood pressure. 2.Maternal pulse. 3.Fetal heart rate. 4.Fetal fibronectin level.

3.Fetal heart rate.

A 28-week-gestation client with intact membranes is admitted with the following findings: Contractions every 5 min x 60 sec, 3 cm dilated, 80% effaced. Which of the following medications will the obstetrician likely order?1.Oxytocin (Pitocin). 2.Ergonovine (Methergine). 3. Magnesium sulfate 4.Morphine sulfate

3. Magnesium sulfate... Magnesium sulfate is a tocolytic agent. It would be appropriate for this medication to be administered at this time.

A nurse is caring for four laboring women. Which of the women will the nursecarefully monitor for signs of abruptio placentae? 1.G3P0020, 17 years of age. 2.G4P2101, cancer survivor. 3.G5P1211, cocaine abuser. 4.G6P0323, 27 weeks' gestation

3.G5P1211, cocaine abuser.... Cocaine is a powerful vasoconstrictive agent. It places pregnant clients at high risk for placental abruption

The childbirth education nurse is evaluating the learning of four women, 38 to 40 weeks' gestation, regarding when they should go to the hospital. The nurse determines that the client who makes which of the following statements needs additional teaching? 1.The client who says, "If I feel a pain in my back and lower abdomen every 5 minutes." 2.The client who says, "When I feel a gush of clear fluid from my vagina." 3.The client who says, "When I go to the bathroom and see the mucus plug onthe toilet tissue." 4.The client who says, "If I ever notice a greenish discharge from my vagina."

3.The client who says, "When I go to the bathroom and see the mucus plug on the toilet tissue."

One hour ago, a multipara was examined with the following results: 8 cm, 50% effaced, and+1 station. She is now pushing with contractions and the fetal head is seen at the vaginal introitus. The nurse concludes that the client is now 1.9 cm dilated, 70% effaced, and+2 station. 2.9 cm dilated, 80% effaced, and +3 station. 3.10 cm dilated, 90% effaced, and+4 station. 4.10 cm dilated, 100% effaced, and+5 station

4. 10 cm dilated, 100% effaced, and +5 station

marked variability

>25bpm

External monitoring uses 2 transducers

Transducer: fetal HR Topotransducer: fetal contractions This is only accurate if the mother and fetus are relatively still

It is 4 p.m. A client, G1P0000, 3 cm dilated, asks the nurse when the dinner tray will be served. The nurse replies 1."Laboring clients are never allowed to eat." 2."Believe me, you will not want to eat by the time it is the dinner hour. Most women throw up, you know." 3."The dinner tray should arrive in an hour or two." 4."A heavy meal is discouraged. I can get clear fluids for you whenever you would like them, though."

4."A heavy meal is discouraged. I can get clear fluids for you whenever you would like them, though."

A woman, 40 weeks' gestation, calls the labor unit to see whether or not she should go to hospital to be evaluated. Which of the following statements by the woman indicates that she is probably in labor and should proceed to the hospital? 1."The contractions are 5 to 20 minutes apart." 2."I saw a pink discharge on the toilet tissue when I went to the bathroom." 3."I have had cramping for the past 3 or 4 hours."

4."The contractions are about a minute long and I am unable to talk through them."

A nurse administers magnesium sulfate via infusion pump to an eclamptic woman inlabor. Which of the following outcomes indicates that the medication is effective? 1.Client has no patellar reflex response. 2.Urinary output 30 cc/hr. 3.Respiratory rate 16 rpm. 4.Client has no grand mal seizures

4.Client has no grand mal seizures... The absence of seizures is an expected outcome related to magnesium sulfate administration

A nurse describes a client's contraction pattern as: frequency every 3 min and duration 60 sec. Which of the following responses corresponds to this description? 1.Contractions lasting 60 seconds followed by a 1-minute rest period. 2.Contractions lasting 120 seconds followed by a 2-minute rest period. 3.Contractions lasting 2 minutes followed by a 60-second rest period. 4.Contractions lasting 1 minute followed by a 120-second rest period.

4.Contractions lasting 1 minute followed by a 120-second rest period.

A woman is scheduled to have an external version for a breech presentation. The nurse carefully assesses the client's chart knowing that which of the following is a contraindication to this procedure? 1.Station-2. 2.38 weeks' gestation. 3.Reactive NST. 4.Previous cesarean section

4.Previous cesarean section...

A nurse determines that a client is carrying a fetus in the vertical lie. The nurse's judgment should be questioned if the fetal presenting part is which of the following? 1.Sacrum. 2.Occiput. 3.Mentum. 4.Scapula.

4.Scapula

A client is on magnesium sulfate for severe preeclampsia. The nurse must notify theattending physician regarding which of the following findings? 1.Patellar and biceps reflexes of+3. 2.Urinary output of 30 cc/hr. 3.Respiratory rate of 16 rpm. 4.Serum magnesium level of 9 gm/dL

4.Serum magnesium level of 9 gm/dL...

Marked variability

is also bad because it is TO MUCH movement

Early Head

Head is becoming compressed during contraction and you are getting an EARLY deceleration

IF THE CORD IS HANGING OUT....

KNEE CHEST POSITION IMMEDIATELY and the nurse takes 2 fingers into the vagina and push up the presenting part to prevent pinching the umbilical cord. You and your hand and the mother will go straight the C-section and you will stay there until the C-section is done

Ultrasound

When: anytime Why: fetal monitoring How--> transabdominal: full bladder required OR transvaginal: full bladder suggested Results: varies depending on test

TRUE LABOR IS WHEN ......

YOU SEE CHANGES IN THE CERVIX!!! ON THE TEST!!!

There should be no observable change in the FHR from the baseline during

a contraction

Station

ischial spines...at zero station once becomes engaged. + 4 on the floor. +5 in the bed. ;measure of the degree of descent through the birth canal. The ischial spines are 0 station.

DYSTOCIA

abnormal or obstructed labor

Late decelerations are considered

an ominous sign.

Ataractics

analgesic potentiators; Visteril and Fenegran= they put it with a pain medicine to help the narcotic work more efficiently

Epidural

anesthesia; can be on a pump or direct injection. HIGHER UP IF a C-section t8-s1 Vaginal delivery T10-t5 SIDE EFFECT: hypotension, need a 1L bolus before hand

FETAL ANATOMY test

anytime between 18-40 weeks. Should be able to detect anomalies. Anatomical anomaly can influence when and how birth can occur.

2 stage of labor-

begins when the cervix is 100% effaced and 10cm dilated. Duration is 15 minutes - 2 hours. Strong expulsive contractions (causes the urge to push). Feelings of anxiety and purpose. FERGUSION REFLEX IS ACTIVATED

port-wine or red fluid

blood (usually seen with abruptio) VERY BAD!!!

Presence of show

bloody show; pink, blood tinged, and sticky

nitrazine paper

checks to make sure fluid isn't urine; amniotic fluid turns paper blue.

Pacemaker point

contraction start at top of uterus and works its way down. It takes both voluntary and involuntary to complete labor. Never start a woman pushing until cervix is completely dilated to 10cm.

Prolapsed Umbilical Cord

cord falls below the presenting part. This happens when the water breaks BUT the baby has not engaged yet; Bradycardia Variable decels. Feel or see the cord

Visible

cord hanging out of the vagina

CRH

corticotropin releasing hormone...sharp increase of this at term. Important to synthesize prostaglandins. Male ejaculate contains prostaglandin. Can onset labor

Common causes of placenta previa:

if you have had 1 you will have another Toyota Previa, C-section previously due to scarring, Multiple babies in one pregnancy, Closely placed pregnancy

Voluntary

doing something to help labor occur

ACTIVE PHASE

duration is from 4-8 cm dilation, lasts between 3-6 hrs. Evidence: Contractions are moderate to strong. Freq: 2-3 mins apart, and last 40-60 secs. Assessment: going in room Q 15-30 mins. Behavior: decreased ability to cope. See epidurals put in at this point. Very restless. Can walk only if membranes have ruptured and baby is engaged at 0 station, or if membranes have not ruptured. Make her comfy as possible

DIF

duration, intensity, frequency

PLACENTA test

during third trimester. Check growth, and placement of the placenta. If see calcium deposits...looks like white flakes. Significant of POST TERM pregnancy. Baby is no longer getting what needs. Placenta previa can be detected

Decelerations

early, late, and variable

Complete rupture

extends through the entire uterine wall into the perennial cavity

Leopold's Maneuver

externally take their hands and feel where baby is or how many babies there are. (Find position of baby)

2 things that EFM look for are

fetal response and fetal compromise

EFM is looking for 2 things

fetal response and fetal compromise

The most common cause of decreased variability of the FHR is

fetal sleep cycles

3rd stage of labor

first few minutes to one hour. If not expelled after an hour is considered RETAINED and have to do D&C to get out. Evidence: still have contractions (clamping down on uterus to prevent hemorrhage, and break off chorionic villi). Assess Q 15mins, check VS, and S&S of hemorrhage. Behavior: mother feels relief

Retained Placenta

if the placenta is delivered more than 30 minutes after delivery or if there is a part of piece left. You must have it removed!

Variability

irregular fluctuations in the baseline FHR of 2 cycles/minute or >

Absent variability

is BAD, fetal death impending

Variability

is what we WANT to see on a fetal heart rate, it is a GOOD thing

Progesterone:

keeps uterus relaxed. Near term hormones drop dramatically and Estrogen takes over which causes contractions.

If the cord is Occult .....

knee to chest position and the mother will be on bed rest

Complete INDIAN breach

legs are crossed, but as though sitting Indian style

Lateral position

may help to rotate the fetus

3rd and 4th tears run risk __________.

loss of bowel control

android

oblong

Braxton hicks

painless contractions

ALPHA-FETOPROTEIN (AFP):

*15-22 weeks. This is a substance produced by the fetal liver, and this should increase as the child gets older. Screening for neural tube defects. Low levels of alpha-fetoprotein associated with down syndrome or spin bifida. High levels you still can have neural tube defects and must be followed up with an ultrasound and then an amniocentesis.

Fetal Growth Test

*When: 20-30 weeks Why: poor maternal weight gain How: Ultrasound Results: Determine growth of fetus. typically done in situations of chronic medical conditions or drug use.

An ultrasound report states, "The fetal head has entered the pelvic inlet." What does the nurse interpret this statement to mean? 1.The fetus has become engaged. 2.The fetal head has entered the true pelvis. 3.The fetal lie is horizontal. 4.The fetus is in an extended attitude.

2.The fetal head has entered the true pelvis

Presentation

96% of births are cephalic 3% of births are breech 1% of births are shoulder

Complete Previa

Cervical Os is completely covered by the placenta (bottom picture) cervix can still dilate. This can cause vaginal bleeding

Soft-Tissue Dystocia

Still in the passageway but it does not include the bone. It could be a fibroid or a placenta previa where the placenta is too low or blocking the cervix or even bladder distention

frank breach

general flexion but instead of cephalic, breach

Baby should be in (attitude) during birth

general flexion: chin is flexed to the abd and the knees and thighs are flexed on the abd (FETAL POSITION)

NORMAL LABOR

if labor was completed with-in 24 hours, a single fetus presents vertex (head down), no complications exist, and the women is near or at term

2) Passenger: ~size of head

fontanels make skull flexible +if baby delivers sinciput or mentum first this can cause baby to look beat up. +want baby to deliver vertex first

2nd stage of labor

from 15 mins to 2 hours. Rectal pressure activates FERGUSON REFLEX.... The presenting part (hope head) has hit the pelvic floor. When it does that the woman has the urge to push. (secondary forces) Assess: every 5-15 mins. Behavior: have sense of purpose Rate of descent: 1 cm per hour nullipara ... multipara 2 cm/hr

TRANSITION PHASE

from 8-10 cm, lasts from 20-40 mins. Evidence: strong contractions, 1.5-2 mins apart. Last 60-90 secs. Assess: every 10-30 mins. Check pain level, baby..etc. Behavior: they are irritable, tired, don't want to be messed with. WILL COMPLAIN OF RECTAL PRESSURE! (baby is crowning.)

duration

from beginning of contraction to the end of the same contraction

IF cardiac issues better to deliver _________ to relieve pressure on the heart

from side

frequency

from the beginning of a contraction to the beginning of the next contraction. (includes the resting period)

attitude

general flexion (fetal position) is best

Periodic change in baseline

those that occur with uterine contractions

Gestational Age Test

*When: 6-40 weeks Why: to determine age of fetus (due date) How: Ultrasound Results: define age of fetus by movement

A nurse is assisting an anesthesiologist who is inserting an epidural catheter. Which of the following positions should the nurse assist the woman into? 1.Fetal position. 2.Lithotomy position. 3.Trendelenburg position. 4.Lateral recumbent position

1.Fetal position

Minimal variability

> undetectable to <5bpm

During delivery, the nurse notes that the baby's head has just been delivered. The nurse concludes that the baby has just gone through which of the following cardinal moves of labor? 1.Flexion. 2.Internal rotation. 3.Extension. 4.External rotation

3.Extension

MEDIOLATERAL

diagonal and less risky

Relaxin

releases in the 3rd trimester: relaxes your pelvis and lets it widen

all fours position

relieves backache

Biophysical Profile

*When: 28 weeks-birth Why: assesses surfactant in lungs; fetal environment, breathing, body movements, HR, & fluid level. Makes sure baby is reacting properly, indicates impending fetal death How: ultrasound Results: Normal BPP indicates fetal wellbeing

Fetal Heart Activity (FHA) Test

*When: 6-7 weeks Why: to check fetal heart How: Ultrasound Results: heart should be beating at 3 weeks

Non-Stress test

*When: anytime but if high risk 30-32 weeks Why: check FHR in relation to fetal movement How: Transabdominally Results:Reactive=good, Nonreactive=bad, Avg FHR=110-160

Contraction Stress Test

*When: anytime; better if done later Why: determine how fetus reacts to stress of contractions How: Oxytocin or nipple stimulation Results: Positive=bad, Negative=good

A nurse is caring for a laboring woman who is in transition. Which of the followingsigns/symptoms would indicate that the woman is progressing into the second stageof labor? Select all that apply. 1.Bulging perineum. 2.Increased bloody show. 3.Spontaneous rupture of the membranes. 4.Uncontrollable urge to push. 5.Inability to breathe through contractions

1.Bulging perineum. 2.Increased bloody show. 4.Uncontrollable urge to push

Which of the following actions would the nurse expect to perform immediately be-fore a woman is to have regional anesthesia? Select all that apply. 1.Assess fetal heart rate. 2.Infuse 1000 cc of Ringer's lactate. 3.Place woman in Trendelenburg position. 4.Monitor blood pressure every 5 minutes for 15 minutes. 5.Have woman empty her bladder

1.Assess fetal heart rate. 2.Infuse 1000 cc of Ringer's lactate 5.Have woman empty her bladder

A woman is in the transition phase of labor. Which of the following comments should the nurse expect to hear? 1."I am so excited to be in labor." 2."I can't stand this pain any longer!" 3."I need ice chips because I'm so hot." 4."I have to push the baby out right now!"

2."I can't stand this pain any longer!"

A low-risk 38-week-gestation woman calls the labor unit and says, "I have to come to the hospital right now. I just saw pink streaks on the toilet tissue when I went tothe bathroom. I'm bleeding." Which of the following responses should the nurse make first? 1."Does it burn when you void?" 2."You sound frightened." 3."That is just the mucus plug." 4."How much blood is there?"

2."You sound frightened."

Which of the following is the appropriate nursing care outcome for a woman who suddenly develops an amniotic fluid embolism during her labor? 1.Client will be infection-free at discharge. 2.Client will exhibit normal breathing function at discharge. 3.Client will exhibit normal gastrointestinal function at discharge. 4.Client will void without pain at discharge

2.Client will exhibit normal breathing function at discharge

A physician has given a nurse a verbal order to apply cricoid pressure. Which of the following is the likely indication for the action? 1.Forceps delivery. 2.Endotracheal tube insertion. 3.Epidural insertion. 4.Third stage of labor

2.Endotracheal tube insertion...

A gravid client at term called the labor suite at 7:00 p.m. questioning whether she was in labor. The nurse determined that the client was likely in labor after the client stated: 1."At 5:00 p.m., the contractions were about 5 minutes apart. Now they're about 7 minutes apart." 2."I took a walk at 5:00 p.m., and now I talk through my contractions easier than I could then." 3."I took a shower about a half hour ago. The contractions seem to hurt more since I finished." 4."I had some tightening in my belly late this afternoon, and I still feel it after waking up from my 2-hour nap."

3."I took a shower about a half hour ago. The contractions seem to hurt more since I finished."

A primigravida is pushing with contractions. The nurse notes that the woman's perineum is beginning to bulge and that there is an increase in bloody show. Which of the following actions by the nurse is appropriate at this time? 1.Report the findings to the woman's health care practitioner. 2.Immediately assess the woman's pulse and blood pressure. 3.Provide encouragement during each contraction. 4.Place the client on her side with oxygen via face mask.

3.Provide encouragement during each contraction.

A 40-week-gestation client has an admitting platelet count of 90,000 mm3 and a hematocrit of 29%. Her lab values 1 week earlier were platelet count 200,000 mm3 and hematocrit 37%. Which additional abnormal lab value would the nurse expect to see? 1.Decreased serum creatinine level. 2.Elevated red blood count (RBC). 3.Decreased alkaline phosphatase. 4.Elevated alanine transaminase (ALT).

4.Elevated alanine transaminase (ALT)....

Which of the following situations in a fully dilated client would warrant the need for a forceps delivery? 1.Maternal history of asthma. 2.Right mentum posterior position and -1 station. 3.Transverse fetal lie. 4.Fetal heart rate of 60 beats per minute

4.Fetal heart rate of 60 beats per minute...

Assessment upon admission

Interview and exam Fetal HR and position Uterine contractions Cervical dilation Presence of show Assessment of fluids Birth plan Psychosocial factors

Hypertonic

The contractions are coming to often and they are ineffective in dilation and effacement CAUSES: New mothers just getting to excited and not relaxing in between contractions. Pitocin in to high a dose can cause this Meds: Sedate them so they will rest and recover energy

Hypotonic

The labor was initially normal in pace and movement but then they suddenly taper off and stop CAUSES: Baby being to big, multiple fetuses, MOST COMMON CAUSE is bladder distention (they need to void through out labor) FIXES: Pitocin administered, have the mother void, change position if the baby is in the wrong position

Lightening:

UTERUS drops at 2 weeks, baby has lightened off of the diaphragm, can breathe but have to pee. Head down before lightening, hopefully.

Sinciput

top of forehead

Marginal

placenta has separated at the Margin (edge), blood passes between the placenta and the uterine wall and escapes out the vagina

Forceps and Vacuum-Assisted

Cervix must be fully dilated, type of pelvis must be none, empty bladder, and membrane ruptured. Forceps and vacuum used to shorten stage 2 IF there is a problem. This will not help with pelvic problem

Prostaglandins

stimulate labor: EXAMPLE Medicine: Cervidil

Evidence shows_________ is not the best position (pushing against gravity with legs in stirrups). The best position to deliver is ______ or _____.

lithotomy (usual) upright or sitting. (walking, sitting, squatting)

green-brown fluid

meconium in fluid (baby is stressed)

There are 2 types of episiotomy

median or midline and mediolateral

Episodic change in baseline

not associated with uterine contractions

Rupture Of Membrane

presenting part is pressing down on "bag of water". R/F infection. No PROM. 80% of women labor will onset within 24 hrs. after ROM. Look for color. Make sure presenting part is engaged. Cord can prolapse

Involuntary contractions called _________ signal the beginning of labor.

primary forces

If the deceleration is variable in nature, the nurse should

put the patient on the left side-lying position to attempt to relieve the pressure off of the cord.

absent variability

range undetectable

Presentation

refers to the part of the fetus that enters the pelvic inlet first. cephalic breech shoulder

Position

relationship of the presenting part with the 4 quadrants of the mother's pelvis. Position is indicated by 3 letter system. *R or L *O for occiput, S for sacrum, M for mentum (chin), and Sc for scapula *location of the part in relation the A (anterior), P (posterior), or (T) transverse

ROA

right occiput anterior

Lithotomy position

usual

Abruptio Placentae Tx

-marginal and central still has blood supply to fetus, just reduced, which may cause decelerations. Monitor fetus for distress -Complete will be automatic C-section immediately -Marginal or central may be able to just do bed rest and fetal monitoring and wait for a bit for the fetus to mature more Only attempt a vaginal birth if the fetus is alive and not in distress or if the fetus has already died

A client just spontaneously ruptured membranes. Which of the following factors makes her especially at high risk for having a prolapsed cord? 1.Breech presentation. 2.Station + 3. 3.Oligohydramnios. 4.Dilation 2 cm

...1.Breech presentation

A client has just entered the labor and delivery suite with ruptured membranes for 2 hours, fetal heart rate of 146, contractions every 5 minutes x 60 seconds, and a history of herpes simplex type 2. She has no observable lesions. After notifying thedoctor of the admission, which of the following is the appropriate action for the nurse to take? 1.Check dilation and effacement. 2.Prepare the client for surgery. 3.Place the bed in Trendelenburg position. 4.Check the biophysical profile results

...1.Check dilation and effacement.

A client has been diagnosed with water intoxication after having received IV oxytocin (Pitocin) for over 24 hours. Which of the following signs/symptoms would the nurse expect to see? 1.Confusion, drowsiness, and vomiting. 2.Hypernatremia and hyperkalemia. 3.Thrombocytopenia and neutropenia. 4.Paresthesias, myalgias, and anemia.

...1.Confusion, drowsiness, and vomiting.

A woman has received Prepidil (dinoprostone gel) for cervical ripening. For which of the following signs/symptoms should the nurse carefully monitor the client? 1.Diarrhea and back pain 2.Hypothermia and rectal pressure. 3.Urinary retention and rash. 4.Tinnitus and respiratory distress

...1.Diarrhea and back pain

A 38-week-gestation woman is in labor and delivery with a painful, board-like ab-domen and progressively larger serial girth measurements. Which of the following assessments is appropriate at this time? 1.Fetal heart rate. 2.Cervical dilation. 3.White blood cell count. 4.Maternal lung sounds

...1.Fetal heart rate.

The nurse is caring for a laboring gravida who is 43 weeks pregnant. For which of the following should the nurse carefully monitor this client and fetus? 1.Late decelerations. 2.Hyperthermia. 3.Hypotension. 4.Early decelerations

...1.Late decelerations.

The physician has ordered oxytocin (Pitocin) for induction for 4 gravidas. In which of the following situations should the nurse refuse to comply with the order? 1.Primigravida with a transverse lie. 2.Multigravida with cerebral palsy. 3.Primigravida who is 14 years old. 4.Multigravida who has type 1 diabetes.

...1.Primigravida with a transverse lie.-->Induction is contraindicated in transverse lie.

Four women request to labor in the hospital bathtub. Which of the women is at in-creased risk from the procedure? 1.Woman during transition. 2.Woman during second stage of labor. 3.Woman receiving oxytocin for induction. 4.Woman with prolonged rupture of the membranes.

4.Woman with prolonged rupture of the membranes

A woman is to receive Prepidil (dinoprostone gel) for labor induction. The nurseshould be prepared to administer the medication via which of the following routes? 1.Intravenously. 2.Orally. 3.Endocervically. 4.Intrathecally

...3.Endocervically

A nurse is teaching a class of pregnant couples the most therapeutic breathing tech-nique for the latent phase of labor. Which of the following techniques did the nurse teach? 1.Alternately panting and blowing. 2.Rapid, deep breathing. 3.Grunting and pushing with contractions .4.Slow chest breathing

4.Slow chest breathing

Immediately after a woman spontaneously ruptures her membranes, the nurse notesa loop of the umbilical cord protruding from the woman's vagina. Which of the following actions should the nurse perform first? 1.Put the client in the knee chest position. 2.Assess the fetal heart rate. 3.Administer oxygen by tight face mask. 4.Telephone the obstetrician with the findings

...1.Put the client in the knee chest position.

A client, 38 weeks' gestation, is being induced with IV oxytocin (Pitocin) for hyper-tension and oligohydramnios. She is contracting q 3 min x60 to 90 seconds. She suddenly complains of abdominal pain accompanied by significant fetal heart bradycardia. Which of the following interventions should the nurse perform first? 1.Turn off the oxytocin infusion. 2.Administer oxygen via face mask. 3.Reposition the patient. 4.Call the obstetrician

...1.Turn off the oxytocin infusion.

A woman, 39 weeks' gestation, is admitted to the delivery unit with vaginal warts from human papillomavirus. Which of the following actions by the nurse is appropriate? 1.Notify the health care practitioner for a surgical delivery. 2.Follow standard infectious disease precautions. 3.Notify the nursery of the imminent delivery of an infected neonate. 4.Wear a mask whenever the perineum is exposed

...2.Follow standard infectious disease precautions

A client is admitted in labor with spontaneous rupture of membranes 24 hours earlier. The fluid is clear and the fetal heart rate is 124 with moderate variability. Which assessment is most important for the nurse to make at this time? 1.Contraction frequency and duration. 2.Maternal temperature. 3.Cervical dilatation and effacement. 4.Maternal pulse rate

...2.Maternal temperature

Which of the following lab values should the nurse report to the physician as being consistent with the diagnosis of HELLP syndrome? 1.Hematocrit 48%. 2.Potassium 5.5 mEq/L. 3.Platelets 75,000. 4.Sodium 130 mEq/L

3.Platelets 75,000...

Immediately prior to an amniotomy, the external fetal heart monitor tracing shows145 bpm with early decelerations. Immediately following the procedure, an internaltracing shows a fetal heart rate of 120 with variable decelerations. A moderateamount of clear, amniotic fluid is seen on the bed linens. The nurse concludes that which of the following has occurred? 1.Placental abruption. 2.Eclampsia. 3.Prolapsed cord. 4.Succenturiate placenta.

...3.Prolapsed cord

The nurse is to intervene when caring for a laboring client whose baby is exhibiting signs of fetal distress. Which of the following actions should the nurse take? 1.Administer oxygen via nasal cannula. 2.Place the client in high Fowler's position. 3.Remove the internal fetal monitor electrode. 4.Increase the intravenous infusion rate

...4.Increase the intravenous infusion rate

A woman, G3P1010, is receiving oxytocin (Pitocin) via IV pump at 3 milliunits/min.Her current contraction pattern is every 3 minutes x45 seconds with moderate intensity. The fetal heart rate is 150 to 160 bpm with moderate variability. Which of the following interventions should the nurse take at this time? 1.Stop her infusion. 2.Give her oxygen. 3.Change her position. 4.Monitor her labor.

...4.Monitor her labor.

A woman, G3P2002, is 6 cm dilated. The fetal monitor tracing shows recurringdeep late decelerations. The woman's doctor informs her that the baby must be de-livered by cesarean section. The woman refuses to sign the informed consent. Which of the following actions by the nurse is appropriate?1.Strongly encourage the woman to sign the informed consent. 2.Prepare the woman for the cesarean section. 3.Inform the woman that the baby will likely die without the surgery. 4.Provide the woman with ongoing labor support

...4.Provide the woman with ongoing labor support

A nurse is monitoring the labor of a client who is receiving IV oxytocin (Pitocin) at 6 mL per hour. Which of the following clinical signs would lead the nurse to stop the infusion? 1.Change in maternal pulse rate from 76 to 98 bpm. 2.Change in fetal heart rate from 128 to 102 bpm. 3.Maternal blood pressure of 150/100. 4.Maternal temperature of 102.4ºF

.2.Change in fetal heart rate from 128 to 102 bpm

A gravid client, G3 P2002, was examined 5 minutes ago. Her cervix was 8 cm dilated and 90% effaced. She now states that she needs to move her bowels. Which of the following actions should the nurse perform first? 1.Offer the client the bedpan .2.Evaluate the progress of labor. 3.Notify the physician. .Encourage the patient to push

.2.Evaluate the progress of labor.

A nurse is caring for women from four different countries. Which of the women ismost likely to request that her head be kept covered throughout her hospitalization? 1.Arabic woman. 2.Chinese woman. 3.Russian woman. 4.Greek woman

1.Arabic woman.

A client who has been diagnosed with severe preeclampsia is being administered magnesium sulfate via IV pump. Which of the following medications must the nurse have immediately available in the client's room? 1.Calcium gluconate. 2.Morphine sulfate. 3.Naloxone (Narcan). 4.Meperidine (Demerol).

1.Calcium gluconate....

A client with a complete placenta previa is on the antepartum clinical unit in prepa-ration for delivery. Which of the following should the nurse include in a teaching session for this client? 1.Coughing and deep breathing. 2.Phases of the first stage of labor. 3.Lamaze labor techniques. 4.Leboyer hydrobirthing

1.Coughing and deep breathing... Because the client will have a cesarean section with anesthesia, the woman should be taught coughing and deep breathing exercises for the postoperative period.

A client with a fetal demise is admitted to labor and delivery in the latent phase of labor. Which of the following behaviors would the nurse expect this client to exhibit? 1.Crying and sad. 2.Talkative and excited. 3.Quietly doing rapid breathing. 4.Loudly chanting songs

1.Crying and sad....

On examination, it is noted that a full-term primipara in active labor is right occipi-toanterior (ROA), 7 cm dilated, and+3 station. Which of the following should the nurse report to the physician? 1.Descent is progressing well. 2.Fetal head is not yet engaged. 3.Vaginal delivery is imminent. 4.External rotation is complete

1.Descent is progressing well.

The nurse is caring for 4 women who are in labor. The nurse is aware that he or she will likely prepare which of the women for cesarean delivery? 1.Fetus is in the left sacral posterior position. 2.Placenta is attached to the posterior portion of the uterine wall. 3.Client is hepatitis B-surface antigen positive. 4.The lecithin/sphingomyelin ratio in the amniotic fluid is 1.5 to 1

1.Fetus is in the left sacral posterior position....

A woman has decided to hire a doula to work with her during labor and delivery. Which of the following actions would be appropriate for the doula to perform? 1.Give the mother a back rub. 2.Assess the fetal heart rate. 3.Check the blood pressure. 4.Regulate the intravenous

1.Give the mother a back rub

A woman, 32 weeks' gestation, contracting every 3 min x 60 sec, is receiving magnesium sulfate. For which of the following maternal assessments is it critical for the nurse to monitor the client? 1.Low urinary output. 2.Temperature elevation. 3.Absent pedal pulses. 4.Retinal edema

1.Low urinary output. ...

A client is in the second stage of labor. She falls asleep immediately after a contrac-tion. Which of the following actions should the nurse perform as a result? 1.Awaken the woman and remind her to push. 2.Cover the woman's perineum with a sheet. 3.Assess the woman's blood pressure and pulse. 4.Administer oxygen to the woman via face mask

2.Cover the woman's perineum with a sheet.

A client's assessments reveal that she is 4 cm dilated and 80% effaced with a fetalheart tracing showing frequent late decelerations and strong contractions every 3 minutes, each lasting 90 seconds. The nursing management of the client should be directed toward which of the following goals? 1.Completion of the first stage of labor. 2.Delivery of a healthy baby. 3.Safe pain medication management. 4.Prevention of a vaginal laceration

2.Delivery of a healthy baby

A woman is in the second stage of labor with a strong urge to push. Which of the following actions by the nurse is appropriate at this time? 1.Assess the fetal heart rate between contractions every 60 minutes. 2.Encourage the woman to grunt during contractions. 3.Assess the pulse and respirations of the mother every 5 minutes. 4.Position the woman on her back with her knees on her chest

2.Encourage the woman to grunt during contractions.

Which of the following physical findings would lead the nurse to suspect that a client with severe preeclampsia has developed HELLP syndrome? 1.+3 pitting edema and pulmonary edema. 2.Epigastric pain and systemic jaundice. 3.+4 deep tendon reflexes and clonus. 4.Oliguria and elevated specific gravity

2.Epigastric pain and systemic jaundice... Epigastric pain and jaundice are reflective of hemolysis of red blood cells and of severe liver damage. These symptoms should make the nurse suspect HELLP syndrome.

The nurse identifies the following nursing diagnosis for a client undergoing anemergency cesarean section: Risk for ineffective individual coping related to emer-gency procedure. Which of the following nursing interventions would be appropriate in relation to this diagnosis? 1.Apply antiembolic boots bilaterally. 2.Explain all procedures slowly and carefully. 3.Administer an antacid per MD orders. 4.Monitor the FH and maternal vital signs

2.Explain all procedures slowly and carefully....

The practitioner is performing a fetal scalp stimulation test. Which of the followingfetal responses would the nurse expect to see?1.Spontaneous fetal movement. 2.Fetal heart acceleration. 3.Increase in fetal heart variability. 4.Resolution of late decelerations

2.Fetal heart acceleration.

A doctor orders a narcotic analgesic for a laboring client. Which of the following situations would lead a nurse to hold the medication? 1.Contraction pattern is every 3 min x 60 sec. 2.Fetal monitor tracing shows late decelerations. 3.Client sleeps between contractions. 4.The blood pressure is 150/90

2.Fetal monitor tracing shows late decelerations....

A client is in the third stage of labor. Which of the following assessments should the nurse make/observe for? 1.Fetal heart assessment after each contraction. 2.Uterus rising in the abdomen and feeling globular. 3.Rapid cervical dilation to ten centimeters. 4.Maternal complaints of intense rectal pressure.

2.Uterus rising in the abdomen and feeling globular.

A multipara, LOA, station+3, who has had no pain medication during her labor, is now in stage 2. She states that her pain is 6 on a 10-point scale and that she wants an epidural. Which of the following responses by the nurse is appropriate? 1."Epidurals do not work well when the pain level is above level 5." 2."I will contact the doctor to get an order for an epidural right away." 3."The baby is going to be born very soon. It is really too late for an epidural." 4."I will check the fetal heart rate. You can have an epidural if it is over 120."

3."The baby is going to be born very soon. It is really too late for an epidural."

A preterm labor client, 30 weeks' gestation who ruptured membranes 4 hours ago,is being given IM dexamethasone (Decadron). When she asks why she is receiving the drug, the nurse replies: 1."To help to stop your labor contractions." 2."To prevent an infection in your uterus." 3."To help to mature your baby's lungs." 4."To decrease the pain from the contractions."

3."To help to mature your baby's lungs."... Decadron is a steroid that hastens the maturation of the fetal lung fields.

A labor nurse is caring for a client, 30 weeks' gestation, who is symptomatic from a complete placenta previa. Which of the following physician orders should the nurse question? 1.Administer bethamethasone (Celestone) 12 mg IM daily times 2. 2.Maintain strict bed rest. 3.Assess cervical dilation. 4.Regulate intravenous (Ringer's lactate: drip rate to 150 cc/hr).

3.Assess cervical dilation...

A 30-year-old G2P0010 in preterm labor is receiving nifedipine (Procardia). Which of the following maternal assessments noted by the nurse must be reported to the health care practitioner immediately? 1.Heart rate of 100 bpm. 2.Wakefulness. 3.Audible rales. 4.Daily output of 2000 cc

3.Audible rales...

A client, G3P2002, 40 weeks' gestation, who has vaginal candidiasis, has just beenadmitted in early labor. Which of the following should the nurse advise the woman? 1.She may need a cesarean delivery. 2.She will be treated with antibiotics during labor. 3.The baby may develop thrush after delivery. 4.The baby will be isolated for at least one day

3.The baby may develop thrush after delivery

The health care practitioner performed an amniotomy 5 minutes ago on a client,G3P1011, 41 weeks' gestation,-4 station, and ROP position. The fetal heart rateis 140 with variable decelerations. The fluid is green tinged and smells musty. The nurse concludes that which of the following situations is present at this time? 1.The fetus is postterm. 2.The presentation is breech. 3.The cord is prolapsed. 4.The amniotic fluid is infected

3.The cord is prolapsed.... It is likely that the cord is prolapsed because the amniotomy was performed when the presenting part was not yet engaged and because variable decelerations are seen on the FH monitor

While performing Leopold's maneuvers on a woman in labor, the nurse palpates a hard round mass in the fundal area, a flat surface on the left side, small objects on the right side, and a soft round mass just above the symphysis. Which of the fol-lowing is a reasonable conclusion by the nurse? 1.The fetal position is transverse. 2.The fetal presentation is vertex. 3.The fetal lie is vertical. 4.The fetal attitude is flexed.

3.The fetal lie is vertical.

The results from a fetal blood sampling test are reported as pH 7.22. The nurse interprets the results as: 1.The baby is severely acidotic. 2.The baby must be delivered as soon as possible. 3.The results are equivocal warranting further sampling. 4.The results are within normal limits

3.The results are equivocal warranting further sampling

A client is in active labor. Which of the following assessments would warrant immediate intervention? 1.Maternal PaCO2of 40 mm Hg. 2.Alpha-fetoprotein values of 2 times normal.3. 3 fetal heart accelerations during contractions. 4.Fetal scalp sampling pH of 7.19

4.Fetal scalp sampling pH of 7.19...

The nurse documents in a laboring woman's chart that the fetal heart is being "assessed via intermittent auscultation." To be consistent with this statement, the nurse, using a Doppler electrode, should assess the fetal heart at which of the fol-lowing times? 1.After every contraction. 2.For 10 minutes every half hour. 3.Periodically during the peak of contractions. 4.For 1 minute immediately after contractions.

4.For 1 minute immediately after contractions.

A woman, G3P2002, 42 weeks' gestation, is admitted to the labor suite for induc-tion. A biophysical profile (BPP) report on the client's chart states BPP score of 6 of 10. The nurse should monitor this client carefully for which of the following? 1.Maternal hypertension. 2.Maternal hyperglycemia. 3.Increased fetal heart variability. 4.Late fetal heart decelerations.

4.Late fetal heart decelerations.... The baby is at high risk for late fetal heart decelerations secondary to a postmature placenta.

To decrease the possibility of a perineal laceration during delivery, the nurse performs which of the following interventions prior to the delivery? 1.Assists the woman into a squatting position .2.Advises the woman to push only when she feels the urge. 3.Encourages the woman to push slowly and steadily. 4.Massages the perineum with mineral oil

4.Massages the perineum with mineral oil

A client is scheduled for an external version. The nurse would expect to prepare which of the following medications to be administered prior to the procedure? 1.Oxytocin (Pitocin). 2.Ergonovine (Methergine). 3.Betamethasone (Celestone). 4.Terbutaline (Brethine).

4.Terbutaline (Brethine).... Terbutaline (Brethine) is a smooth muscle relaxing agent. It would be administered prior to an external version.

The nurse is interpreting the results of a fetal blood sampling test. Which of the following reports would the nurse expect to see? 1.Oxygen saturation of 99%. 2.Hgb of 11 gm/dL. 3.Serum glucose of 140 mg/dL. 4.pH of 7.30

4.pH of 7.30

Malpresentation

fetus presenting itself in the wrong way. Brow, breech, shoulder, face are all different presentations. C-section required

1st stage of Labor:

latent phase, active phase, transition phase

___________ is an early sign to Hypoxia

tachycardia...>160 b/m

Effleurage

taking fingers and feathering massage of contracting uterus

Episiotomy is worse on your body than a tear, because .......

the body has torn where it needs to only.

Ferguson reflex

the urge to bear down because the baby is ready to come out. FEELS LIKE THEY NEED TO SHIT

If the baby is Malpresentation

they can try to move the baby with VERSION from the outside of the body. It is high risk and painful. This must be done before fetus engages. MUST BE 36 WEEKS as this may cause early labor and must have a non reactive stress test. -->SQ Terbutaline, which helps prevent contractions which mashing the pelvis like pizza dough

4 categories of variability

undetected, minimal variability (<5 bmp), moderate variability (6-25 bpm), and marked variability (>25 bpm)

4th stage of labor

up to 2 hours after the delivery of the placenta.... Hormones are changing

Pudendal Block

used during an Episiotomy, anesthesia is injected directly into the area around the vagina

Incomplete rupture

uterus has ruptured but has not extended into the cavity, the problem with this is you may not see if at first and then the mother is bleeding internally and may not show signs until SHOCK

Dick-Read Method

woman sleeps trough contractions; focus on abdomen


Conjuntos de estudio relacionados

NUR 3306 Final Study Guide: Quizzes 1-4

View Set

Language and Thought People to Know

View Set

Module 10 - Civil Procedure 3 (Section 4)

View Set

PrepU Medical-Surgical Nursing Chapter 42: Obesity

View Set

module 11 - Aging by dental exam

View Set

Chapter 14: Assessing Skin, Hair, and Nails

View Set

section 12 unit 4: Qualifying the Title

View Set